2013 AIME I Problems/Problem 8

Revision as of 18:19, 16 March 2013 by MSTang (talk | contribs)

Problem 8

The domain of the function f(x) = arcsin(log$_{m}$(nx)) is a closed interval of length $\frac{1}{2013}$ , where $m$ and $n$ are positive integers and $m>1$. Find the remainder when the smallest possible sum $m+n$ is divided by 1000.


Solution

The domain of the arcsin function is $[-1, 1]$, so $-1 \le$ log_{m}(nx) \le 1$.$\frac{1}{m} \le nx \le m$$ (Error compiling LaTeX. Unknown error_msg)\frac{1}{mn} \le x \le \frac{m}{n}$$ (Error compiling LaTeX. Unknown error_msg)\frac{m}{n} - \frac{1}{mn} = \frac{1}{2013}$$ (Error compiling LaTeX. Unknown error_msg)n = 2013m - \frac{2013}{m}$For$n$to be an integer,$m$must divide$2013$, and$m > 1$. To minimize$n$,$m$should be as small as possible because increasing$m$will decrease$\frac{2013}{m}$, the amount you are subtracting, and increase$2013m$, the amount you are adding; this also leads to a small$m$which clearly minimizes '$m+n$.

We let$ (Error compiling LaTeX. Unknown error_msg)m$equal 3, the smallest factor of$2013$that isn't$1$..$n = 2013*3 - \frac{2013}{3} = 6039 - 671 = 5368$$ (Error compiling LaTeX. Unknown error_msg)m + n = 5371$, so the answer is$\boxed{371}$.